How can you use the double number line diagram to find what percent is of ​? x y graph Question content area bottom Part 1 Each mark on the top number line increases by enter your response here​, so the ▼ third second fourth first mark is . Each mark on the bottom number line increases by enter your response here​%. ​So, enter your response here​% of is . ​(Type whole​ numbers.)

Answers

Answer 1

Here are the steps to use the double number line diagram to find a percentage:

Identify the two sets of values: Draw a double number line diagram: Determine the total value: Determine the known value:Use the double number line to find the unknown value: Calculate the percentage:

How to explain the information

It should be noted that the double number line diagram is a visual tool that is used to represent two equivalent sets of values, and it can be useful to find percentages in a given context.

In this case, the steps are given above.

Learn more about number line

https://brainly.com/question/24644930

#SPJ1


Related Questions

Please help asap!!!!! I'm confused

Answers

The area of the parallelogram is 8.5 square miles. This is found by multiplying the length of one of the parallel sides, 2 miles, by the height, which is given as 4 1/4 miles.

To find the area of a parallelogram, we can multiply the length of one of its parallel sides by the length of its perpendicular height. Therefore, to find the area of this parallelogram, we need to determine its height.

We are given that one of the parallel sides has a length of 4 1/4 mi and the other has a length of 2 mi. We are also given that the length of the perpendicular on one of the parallel sides is 4 1/4 mi, which means that this is the height of the parallelogram.

So, the area of the parallelogram is

Area = base x height

Area = 2 mi x 4 1/4 mi

Area = 8 1/2 mi²

Therefore, the area of the parallelogram is 8 1/2 square miles or 8.5 square miles.

To know more about Area of parallelogram:

https://brainly.com/question/19187448

#SPJ1

use the equation 1/5 +s =32/40

Answers

The required solution to the equation 1/5 + s = 32/40 is s = 3/5.

To solve the equation 1/5 + s = 32/40 for s, we can begin by subtracting 1/5 from both sides to isolate s:

1/5 + s = 32/40

s = 32/40 - 1/5

We need a common denominator to combine the fractions on the right side of the equation. The least common multiple of 5 and 40 is 40, so we can convert both fractions to have a denominator of 40:

s = (32/40) - (8/40)

s = 24/40

Simplifying the fraction 24/40 by dividing both the numerator and denominator by their greatest common factor, which is 8, we get:

s = 3/5

Therefore, the solution to the equation 1/5 + s = 32/40 is s = 3/5.

Learn more about simplification here:

https://brainly.com/question/12501526

#SPJ1

A geography teacher assigns each student to write a report about one of the first 13 colonies. Students select the name of a colony by "blindly" drawing a colony's name from a bag. Once a colony has been drawn, it is not replaced.

What is the probability that the first student selects Pennsylvania and the second student selects Virginia? Round your answer to the nearest hundredth of a percent.

Answers

The probability that the first student selects Pennsylvania and the second student selects Virginia to the nearest hundredth of a percent is 0.64%.

Probability problem

The probability that the first student selects Pennsylvania is 1/13. Once Pennsylvania has been drawn, there are only 12 colonies left in the bag, so the probability that the second student selects Virginia is 1/12.

To find the probability that both events occur, we multiply the probabilities:

(1/13) x (1/12) = 1/156

To express this probability as a percentage, we multiply by 100:

(1/156) x 100 ≈ 0.64%

Therefore, the probability that the first student selects Pennsylvania and the second student selects Virginia is approximately 0.64%.

More on probability can be found here: https://brainly.com/question/30034780

#SPJ1

Which equation represents a direct variation?
Oy - 4x = 8
Oy + 2 = 7x
Oy - 3x = 0
O y = 5x - 2

Answers

The only equation that represents a direct variation is: y - 3x = 0

How to identify direct variation?

Direct Variation is defined as the relationship that exists between two variables in which one is a constant multiple of the other. For example, when one variable changes the other, then they are said to be in proportion. If b is directly proportional to a the equation is of the form b = ka (where k is a constant).

Making y the subject in each of the options gives:

A) y = 4x + 8

B) y = 7x - 2

C) y = 3x

D) y = 5x - 2

Looking at them, the only one where there is a relationship that exists between two variables in which one is a constant multiple of the other is option C

Read more about Direct variation at: https://brainly.com/question/6499629

#SPJ1

Keng adds a 3-inch-wide frame around all sides of his canvas. ​

Answers

Answer:

That statement describes an action taken by Keng to add a 3-inch-wide frame around all sides of his canvas, likely for artistic or aesthetic purposes. This would result in the canvas being extended by 3 inches in each direction, effectively increasing its overall dimensions and adding a border or frame around the original artwork. The purpose and effect of adding a frame may vary depending on the artistic intention of the artist, but it is a common practice in art and design to enhance the presentation and visual appeal of the artwork.

Step-by-step explanation:

An axon is a

long, tubelike structure extending from a neuron's cell body

branch-like fiber extending in clusters from a neuron's cell body

neuron's cell body

messenger of the nervous system.

Answers

An axon is a long, tubelike structure extending from a neuron's cell body branch-like fiber extending in clusters from a neuron's cell body messenger of the nervous system is true.

We have,

An axon is a long, tubelike structure that extends from a neuron's cell body.

It is a specialized fiber that carries electrical impulses, known as action potentials, away from the cell body and towards the axon terminals.

Axons are covered by a fatty substance called myelin, which insulates and protects the axon and helps to speed up the transmission of signals.

At the end of the axon, there are small branches called axon terminals, which release chemicals called neurotransmitters into the synapse, the small gap between the axon terminals and the dendrites of the next neuron in the circuit.

This allows the electrical signal to be converted into a chemical signal and transmitted to the next neuron, continuing the chain of communication in the nervous system.

Thus,

An axon is a long, tubelike structure extending from a neuron's cell body branch-like fiber extending in clusters from a neuron's cell body messenger of the nervous system is true.

Learn more about axons here:

https://brainly.com/question/28234182

#SPJ1

Margarita fue a la tienda y compro una cartera y unos jeans por un
total de RD$3,250. Sabiendo que las cartera excede al jeans en
RD$970, ¿Cuántos pago margarita por cada artículo?
Cartera = RDS
Jeans = RD$
Escribir las respuestas numéricas y sin comas.
OK

Answers

The solution is , price of jeans = RD$ 1140 and, price of  handbag =

RD$ 2110.

Here, we have,

given that,

Margarita went to the store and bought a bag and some jeans for a total of RD$3,250.

Knowing that the handbag exceeds the jeans by RD$970,

now, we have to find that, how many do she pay for each article.

let, price of jeans = RD$ x

so, price of  handbag = RD$ (x +970)

ATQ, we get,

RD$ x + RD$ (x +970) = RD$3,250

or, RD$ 2x + 970 = RD$3,250

or, RD$ 2x = RD$ 2280

or, x = RD$ 1140

Hence, price of jeans = RD$ 1140 and, price of  handbag = RD$ 2110.

To earn more on addition click:

brainly.com/question/29560851

#SPJ1

Helppp I need the ending numbers ?

Answers

The solution is, the simplification of the equation is :

4x^2 - 4 = (2x -2 ) (2x+2)

Here, we have,

given that,

the expression is:

4x^2 - 4

now, we have to simplify this.

we know that ,

The a^2 - b^2 formula is also known as "the difference of squares formula". The a square minus b square is used to find the difference between the two squares without actually calculating the squares.

It is one of the algebraic identities.

It is used to factorize the binomials of squares.

The a^2 - b^2 formula is given as:

a^2 - b^2 = (a - b) (a + b).

so, we have,

4x^2 - 4

= (2x)^2 - 2^2

= (2x -2 ) (2x+2)

Hence, The solution is, the simplification of the equation is :

4x^2 - 4 = (2x -2 ) (2x+2)

To learn more on Equation:

brainly.com/question/10413253

#SPJ1

A belt runs a pulley at 80 revolutions per minute. Find the angular velocity of the pulley in radians per second.

Answers

A belt runs a pulley at 80 revolutions per minute.

(a) To find the angular speed of the pulley in radians per second, we can use the formula

ω = 2πf

Where

ω = angular speed in radians per second

f = frequency in Hertz

We know that the pulley rotates at a rate of 80 revolutions per minute. To convert this to a frequency in Hertz, we can divide by 60 seconds per minute

f = 80 rev/min ÷ 60 min/s = 4/3 Hz

Now we can plug in this value for f into the formula

ω = 2πf = 2π(4/3) = 8π/3 rad/sec

Therefore, the angular speed of the pulley is 8π/3 radians per second.

(b) To find the linear speed of the belt in centimeters per second, we can use the formula

v = rω

Where

v = linear speed in centimeters per second

r = radius of the pulley in centimeters

ω = angular speed in radians per second

We know the radius of the pulley is given in centimeters. Let's assume it is r cm. We just found the angular speed to be 8π/3 radians per second. Now we can plug in these values into the formula

v = rω = r(8π/3) = (8π/3)r cm/s

Therefore, the linear speed of the belt is (8π/3)r centimeters per second.

The given question is incomplete and the complete question is ''A belt runs a pulley at 80 revolutions per minute. Find the angular velocity of the pulley in radians per second. (b)Find the linear speed of the belt in centimeters per second''.

To know more about revolutions here

https://brainly.com/question/29707118

#SPJ1

Length of the Glass = ? if length of 6 glasses is 34 cm and length of 2 glasses 19 cm find length of 1 glass​

Answers

The length of the glass is L = 5.67 cm

Given data ,

Let the length of one glass be "x" cm

According to the given information, the length of 6 glasses is 34 cm.

So, the combined length of 6 glasses is 6x cm, which is equal to 34 cm.

Similarly, the length of 2 glasses is 19 cm.

So, the combined length of 2 glasses is 2x cm, which is equal to 19 cm.

6x = 34 be equation (1)

2x = 19 be equation (2)

Now we can solve these equations to find the value of "x", which represents the length of one glass.

On dividing equation 1 by 6, we get:

x = 34/6

x ≈ 5.67 cm

Hence , the length of one glass is approximately 5.67 cm

To learn more about equations click :

https://brainly.com/question/19297665

#SPJ1

Alice finds shirts on sale for $18.99.She buys twelve how much money does she spend?

Answers

Answer:

Well, if Alice buys 12 shirts which each cost $18.99 the equation would be 18.99 * 12 which = 227.88

Alice spent $227.88 on 12 shirts

Step-by-step explanation:

Answer:

Well, if Alice buys 12 shirts which each cost $18.99 the equation would be 18.99 * 12 which = 227.88

Step-by-step explanation:

Give the rectangular coordinates for the point

Answers

The rectangular coordinates from the polar coordinates are: (-4√2, -4√2)

How to convert polar coordinates to rectangular coordinates?

The steps to  to convert polar coordinates to rectangular coordinates are:

Step 1: Find the x -coordinate for the rectangular coordinate form of the point by using the equation x = r cos(θ)

Step 2: Find the y -coordinate for the rectangular coordinate form of the point by using the equation y = rsin(θ)

Step 3: Write the rectangular coordinates as (x,y) using the results from steps 1 and 2.

We are given the polar coordinates as (8, 225°)

Thus:

x-coordinate of rectangular coordinate = 8 cos 225 = -4√2

y-coordinate of rectangular coordinate = 8 sin 225 = -4√2

Read more about rectangular coordinates at: https://brainly.com/question/29155884

#SPJ1

Help with this page :)

Answers

17. 15 degrees

18. ABC

19. 5.88

20. 11.59

let me know if you'd like an explanation

construct a binomial whose gcf is 7a^3

Answers

The binomial whose greatest common factor is 7a³ is 14a⁴ + 35a³.

Given, the a³ is not common in A and C.

Now, in 14a⁴ + 35a³

14a⁴ = 2 x 7 x a³ x a

35a³ = 7 x 5 x a³

So, the common factors of 14 and 35 is 7.

Then, 14a⁴ + 35a³

= ( 2 x 7 x a³ x a) + (  5 x 7 x a³)

= 7a³ (2a + 5)

Learn more about greatest common factor here:

brainly.com/question/11221202

#SPJ1

In Brian's grade, 440 students are enrolled in health and 60 students are not. What percentage of the students in the school are enrolled in health?

Answers

88% of the students in the school are enrolled in health.

We have,

Students enrolled in health = 440

Students not enrolled = 60

Total students = 500

So, x% of 500 = 440

x/100 (500) = 440

5x = 440

x = 88%

Thus, 88% of the students in the school are enrolled in health.

Learn more about Percentage here:

https://brainly.com/question/29306119

#SPJ1

geometry pls help fast 13 and 14

Answers

Answer:

13: (2,3) 14: A. Yes, 90, 180, 270 B. No C. No

Step-by-step explanation:

13. Right: -2+4, Down: 5-2

14. If it can be rotated and be similar, it has rotational.

Forty people were asked their favorite kind of pizza. Thirty percent of the people surveyed chose sausage. How many people preferred sausage?

Answers

To find out how many people preferred sausage, you can start by multiplying the percentage who chose sausage, which is 30%, by the total number of people surveyed, which is 40.

So, 30% of 40 is:

0.30 x 40 = 12

Therefore, 12 people preferred sausage.

Answer: 12 people

Step-by-step explanation:

0.30 x 40 = 12

An insurance company offers an ordinary annuity that earns 6.5% interest compounded annually. A couple plans to make equal annual deposits into this account for 30 years and then make 20 equal annual withdrawals of €25,000, reducing the balance of the account to zero.

(i) Compute the value of the fund based on the withdrawals required. [5 marks]

(ii) Compute the amount of each deposit needed in order to maintain the fund. [5 marks]

(iii) Compute the total interest earned over the entire 50 years. [5 marks]​

Answers

Answer:

(i) To compute the value of the fund based on the withdrawals required, we can use the formula for the future value of an annuity due:

FV = P * ((1 + r)^n - 1) / r) * (1 + r)

where FV is the future value of the annuity, P is the annual payment, r is the interest rate per period, n is the total number of periods, and the extra (1 + r) factor is because the payments are made at the beginning of each period.

In this case, P = €25,000, r = 0.065, n = 20. We want to find the future value at the end of the 20-year period:

FV = 25000 * ((1 + 0.065)^20 - 1) / 0.065) * (1 + 0.065)

FV ≈ €743,704.96

Therefore, the value of the fund based on the withdrawals required is approximately €743,704.96.

(ii) To compute the amount of each deposit needed in order to maintain the fund, we can use the formula for the present value of an ordinary annuity:

PV = P * ((1 - (1 + r)^(-n)) / r)

where PV is the present value of the annuity, P is the annual payment, r is the interest rate per period, and n is the total number of periods.

In this case, PV = €743,704.96, r = 0.065, n = 20. We want to find the annual payment:

PV = P * ((1 - (1 + 0.065)^(-20)) / 0.065)

P ≈ €22,630.53

Therefore, the amount of each deposit needed in order to maintain the fund is approximately €22,630.53.

(iii) To compute the total interest earned over the entire 50 years, we can subtract the total deposits from the total withdrawals, and then subtract the initial balance. The total deposits are the annual deposit amount times the number of years (30), and the total withdrawals are the annual withdrawal amount times the number of years (20). The initial balance is the present value of the annuity that we calculated in part (ii).

Total deposits = €22,630.53 * 30 = €678,915.90

Total withdrawals = €25,000 * 20 = €500,000

Initial balance = €743,704.96

Total interest earned = Total withdrawals - Total deposits - Initial balance

Total interest earned = €500,000 - €678,915.90 - €743,704.96

Total interest earned ≈ -€922,620.86

Note that the negative sign indicates that the insurance company actually earned interest on this annuity, rather than the couple earning interest on their investment. This is because the withdrawals are greater than the deposits, and the interest rate earned by the insurance company is greater than the interest rate paid to the couple.

Step-by-step explanation:

Need help. This please

Answers

The domain of the quadratic function in this problem is given as follows:

All real values.

How to obtain the domain of the function?

The domain of a function is the set of all the possible input values that can be assumed by the function.

On the graph, the domain of the function is given by the values of x of the function.

A quadratic function has no restrictions on the domain, hence it is defined by all the real values.

More can be learned about the domain of a function at https://brainly.com/question/10687170

#SPJ1

how do you find the net of a rectangular prism

Answers

Answer:

The formula looks like this:

Surface Area = 2 l w + 2 l h + 2 h w ,

where SA = surface area, l = length, w = width, and h = height. In the rectangular prism net above, l = 8 inches, w = 5 inches, and h = 3 inches. Simply put these numbers into the formula and solve for surface area.

The following blueprint of a kitchen has dimensions of 7 inches by 7 inches. The island has been highlighted in red.


The island's actual dimensions are 3 1/2 feet by 1 3/4 feet. If the scale of the blueprint is 1 inch = 2 feet, what are the dimensions of the island on the blueprint?

Answers

The dimensions of the island on the blueprint are 14 inches by 3.5 inches.

We have,

The actual dimensions of the island are 3 1/2 feet by 1 3/4 feet.

We need to find the dimensions of the island on the blueprint, given that the scale of the blueprint is 1 inch = 2 feet.

To convert the actual dimensions to the dimensions on the blueprint, we need to use the scale factor of 1 inch = 2 feet.

We can set up a proportion to relate the actual dimensions to the dimensions on the blueprint:

Actual dimension/blueprint dimension = scale factor

Let x be the length of the island on the blueprint.

Then we can set up the following proportion:

3.5 feet / (1.75 feet)

= x inches / 7 inches

Simplifying,

2 = x / 7

Multiplying both sides by 7, we get:

x = 14 inches

The length of the island on the blueprint is 14 inches.

Similarly, we can find the width of the island on the blueprint:

1.75 feet / 3.5 feet

= y inches / 7 inches

Simplifying, we have:

0.5 = y / 7

Multiplying both sides by 7, we get:

y = 3.5 inches

The width of the island on the blueprint is 3.5 inches.

Thus,

The dimensions of the island on the blueprint are 14 inches by 3.5 inches.

Learn more about expressions here:

https://brainly.com/question/3118662

#SPJ1

Find the angle between the pair of vectors to the nearest tenth of the degree

Answers

The value of angle between the two vectors is 86⁰.

What is the angle between the two vectors?

The value of angle between the two vectors is calculated as follows;

tan θ = vy/vx

where;

vy is the sum of the vertical directionvx is the sum of vectors in horizontal direction

( -8, 9), (-9, -6)

vy = (-6 - 9) = -15

vx = (-9 + 8) = -1

tan θ = ( -15 ) / ( -1 )

tan θ = 15

The value of θ is calculated  by taking arc tan of the fraction,;

θ = tan ⁻¹ ( 15 )

θ =  86⁰

Learn more about direction here: https://brainly.com/question/30318208

#SPJ1

What is the value of z?

Answers

Answer: z = 8

Step-by-step explanation:

The diagram shows us that 8z + 3z + 2 = 90

so we can say that: 11z = 88

therefore z = 8.

Note: diagrams can be misleading! this diagram technically shows us that 64 < 26!

Brooklyn and Matthew both recently got a job and want to start a savings account to earn interest on money they save. Brooklyn's paycheck for was for $625 net pay-after taxes. the bank her parents use offer savings account compounded monthly with an interest rate of 2.5%. Matthews paycheck was also for $446 net pay. his parents know of a savings account that earns 4% compounded quarterly.
a. use the compounding interest rate formula to identify who will have more money after two years (assuming no additional deposits or withdrawals are made)
b. use the Desmos graphing calculator to graph Brooklyn's equation and Matthew’s equation on the same graph. identify the points of intersection and interpret the results.

WHAT MISTAKES DID THE STUDENTS MAKE??

Answers

After two years, Brooklyn will have more money in her savings account than Matthew.

Brooklyn's savings account grows at a slower rate than Matthew's initially but eventually catches up and surpasses Matthew's account due to the higher interest rate and monthly compounding.

a.

[tex]A = P(1 + r/n)^{nt}[/tex]

where A is the final amount, P is the initial principal (or net pay), r is the interest rate (as a decimal), n is the number of times the interest is compounded per year, and t is the number of years.

For Brooklyn saving account,

P = $625, r = 0.025, n = 12 (monthly compounding), and t = 2.

A = $625(1 + 0.025/12)^(12*2) = $686.71

For Matthew saving account,

P = $446, r = 0.04, n = 4 (quarterly compounding), and t = 2.

A = $446(1 + 0.04/4)^(4*2) = $506.84

b.

To graph Brooklyn's equation and Matthew's equation on the same graph, we can use the following equations:

Brooklyn's equation: A = 625(1 + 0.025/12)^(12t)

Matthew's equation: A = 446(1 + 0.04/4)^(4t)

Now,

The resulting graph will show the growth of each saving account over time.

The points of intersection on the graph represent the times when the two savings accounts have the same amount of money.

From the graph, we can see that the two accounts intersect at around 11 months and 23 months.

We can say that Brooklyn's savings account grows at a slower rate than Matthew's initially, but eventually catches up and surpasses Matthew's account due to the higher interest rate and monthly compounding.

Thus,

After two years, Brooklyn will have more money in her savings account than Matthew.

Brooklyn's savings account grows at a slower rate than Matthew's initially but eventually catches up and surpasses Matthew's account due to the higher interest rate and monthly compounding.

Learn more about saving accounts here:

https://brainly.com/question/13155407

#SPJ1

Find the value of the following expression and round to the nearest integer:

Answers

The value of the Expression is 610, 919.

We have,

Expression : [tex]\sum_{n=0}^{61[/tex] 700 (1.07[tex])^{n+1[/tex]

The expression is a summation formula, representing the sum of a series of values.

Here r= 1.07 and n is number of terms.

So, [tex]\sum_{n=0}^{61[/tex] 700 (1.07[tex])^{n+1[/tex]

= 700 (1) +  [tex]\sum_{n=0}^{61[/tex] 700 (1.07[tex])^{n[/tex]

= 700 + 700 ([tex]r^{61[/tex] - 1)/ (r-1)

= 700 + 700 ([tex](1.07)^{61[/tex]-1)/ (0.07)

= 700+ 700 (871.7428)

= 610, 919

Learn more about Summation here:

https://brainly.com/question/29103782

#SPJ1

Is this right. Don’t know the difference between the stratified and systematic random. Pic below

Answers

The difference between the stratified and systematic random sampling is explained below.

Systematic random sampling is a method where every Kth person of the population is chosen to be part of the sample, whereas stratified random sampling is a method where the population is first divided into subgroups, and then drawing a simple random sample from each subgroup.

Learn more about stratified and systematic random sampling click;

https://brainly.com/question/28963265

#SPJ1

Solve for m. m/2 - squareroot 5 = 3

Answers

Answer:

We can start by isolating the variable term on one side of the equation and moving all other terms to the other side.

First, we'll add the square root of 5 to both sides of the equation:

m/2 - sqrt(5) + sqrt(5) = 3 + sqrt(5)

Simplifying:

m/2 = 3 + sqrt(5)

Next, we'll multiply both sides of the equation by 2:

m = 2(3 + sqrt(5))

Simplifying:

m = 6 + 2sqrt(5)

Therefore, the value of m is 6 + 2sqrt(5).

Step-by-step explanation:

Helpppp I need helppp

Answers

In this equation:

The blank would be 2 and x=1

You can use the notation P(A), read “the probability of event B, given event A” to write a

A. Probability distribution
B. Frequency table
C. Conditional probability
D. Cumulative probability

Answers

You can use the notation P(A), read “the probability of event B, given event A” to write a conditional probability. The correct answer is C.

Conditional probability refers to the probability of one event occurring given that another event has already occurred. In this case, we are interested in the probability of event B occurring given that event A has already occurred, and we can represent this using the notation P(B|A), where '|' means 'given'.

For example, let's say we are interested in the probability of getting a head on a coin toss (event B), given that the coin was flipped and landed on heads (event A). We could represent this using the notation P(B|A). The value of P(B|A) would be 1, because if the coin already landed on heads, then the probability of getting a head on the next flip is certain.

Conditional probability is an important concept in probability theory and is often used in real-world applications, such as predicting the likelihood of a disease given certain symptoms, or the probability of an event occurring given certain conditions.

The correct answer is C.

To learn more about probability click on,

https://brainly.com/question/29259732

#SPJ1

Determine if the statement is true or false.

Angle AEC and DEB are complementary angles and therefore add up to 180 degrees.

1. True
2. False

Answers

The statement "Angle ∠AEC and ∠DEB are complementary angles" is false.

Supplementary angle - Two angles are said to be supplementary angles if their sum is 180 degrees.

Complementary angle - Two angles are said to be complementary angles if their sum is 90 degrees.

Angle ∠AEC and ∠DEB are supplementary angles because the sum is 180 degrees.

Thus, the given statement is false.

More about the angled link is given below.

https://brainly.com/question/15767203

#SPJ1

Other Questions
what helps drive the east-west circuit of air in the tropics? multiple choice question. a reverse ekman spiral as the wind is pushed by the north-south water currents below gravitational attraction to the moon as it makes its passage across the sky adiabatic warming of the rising air along the equator the formation of warm pools and the rising air found above them -- a method of stocking items in which parts are placed in any space that is empty when they arrive at the storeroom or warehouse. this method uses less space but is more dependent on inventory tracking software. -- the systems used to manage business processes within a storage or distribution facility, including receiving, putaway, picking, shipping, and inventory cycle counts. David has twenty dimes (d) and quarters (q). These coins total $2.75. How many of each type of coin does he have? Write a systems of equations to model this scenario. Show your work and state how many of each type of coin he has. Enter your systems of equations, your work, and your statement of how many of each type coin David has below. In literature, unreliable narrators are narrators who are not credible or trustworthy. True False The unit you work with is leaving a forward area rearming/refueling point and has unused ammunition. The ammunition should be Which one is not one of the alternatives in building the Web site ? a.completely outhouse b.completely inhouse d.completely outsourced e.mixed responsibility Rachel Carson's book, ________, pointed out the dangers posed to the natural environments and humans by pesticides.A) The Population BombB) NatureC) Silent SpringD) Man and NatureE) Walden The three general types of media schedules are make me a fortune cookie flowchart with photo PLEASE HELP HURRY FAST PLEASE THANK YOU!4. Find the rational roots of x^4+5x^3+7x^2-3x-10=0A. -2,1B. 2,1C. -2,-1D. 2,-15. Find all the zeros of the equation 3x^2-4=-x^4A. 1,2iB. -1,-2iC. 1,-1,2i,-2i,0D. 1,-1,2i,-2i6. What is a polynomial function in standard form with zeros 1,2,-2, and -3?A. x^4+2x^3+7x^2-8x+12B. x^4+2x^3-7x^2-8x+12C. x^4+2x^3-7x^2+8x+12D. x^4+2x^3+7x^2+8x+127. Which correctly describes the roots of the following cubic question.x^3-3x^2+4x-12=0A. three real roots, each with a different valueB. one real root and two complex rootsC. three real roots, two of which are equal in valueD. two real roots and one complex root8. What is the solution of 5^3x=900. Round your answer to the nearest hundredthA. 1.24B. 1.41C. 4.23D. 0.699. x= 1,2,4,6,8,10,11 y= 0,-1,0,4,8,9,8 Table x= 2,5,12,20 y= 30,12,5,3 A. direct variation; y=15xB. inverse variation; y=60/xC. direct variation; y=2x+2D. neither11. A drama club is planning a bus trip to New York City to see a Broadway play. the cost per person for the bus rental varies inversely as the number of people going on the trip. It will cost $30 per person if 44 people go on the trip. How much will it cost per person if 55 people go on the trip. Round your answer to the nearest cent if necessary A. $48.00B. $12.50C. $24.00D. $33.0012. What is the simpler form of the radical expression? 4sqrt2401x^12y^16A. 49|x^9|y^16B. 49x^9|y^16|C. 7|x^3|y^4D. 7x^3|y^4|13. Simplify. 125^1/3 A. 125B. 5C. 25D. sqrt12514. Graph the function. y=sqrtx+315. An initial population of 293 quail increases at an annual rate of 6%. Write an exponential function to model the quail population. What will the approximate population be after 4 years?A. f(x)=(293 x 1.06)^x; 930B. f(x)=293(0.06)^x; 379C. f(x)=293(1.06)^x; 370D. f(x)=293(6)^x; 19016. Evaluate the logarithm log3 2187A. 5B. 6C. 7 D. -717. Estimate the value of the logarithm to the nearest tenthlog4 22A. 2.2B. 0.4C. -0.7D. 3.218. Solve 1n 4 + 1n (3x)=2. Round your answer to the nearest hundredth A. 1.13B. 0.18C. 1.41D. 0.6219. Write an equation for the translation of y= 4/x that has the asymptotes x=7 and y=6.A. y= 4/x-6 +7B. y= 4/x+7 +6C. y= 4/x-7 +6D. y= 4/x+6 +720. What is the graph of the rational function?y=(x-5)(x-3)/(x+4)(x-4)21. What are the points of discontinuity?y=(x-3)/x^2-12x+27A. x=4,x=9,x=1B. x=-9,x=-3C. x=-9,x=-3,x=-1D. x=9,x=322. What is the quotient 6-x/x^2+2x-3/x^2-4x-12/x^2+4x+3 in simplified form? State any restrictions on the variable.23. Simplify the complex fraction x+3/ 1/x+1/x+3A. x^2/2x+3B. x^2/2xC. x^2/x+3D. x^2+2x+3/2x+324. Simplify the differencex^2-x-56/x^2+6x-7 - x^2+2x-15/x^2+9x+20A. x-71/(x-1)(x+4)B. 2x^2-8x-29/(x-1)(x+4)C. -8x-35/(x-1)(x+4)D. -35/(x-1)(x+4)25. Solve the equation 1/x-3+1/x+5=1/3A. x=1,x=7B. x=3,x=-5C. x=-3,x=1D. x=-3,x=7 The market for beignets is competitive. Which of the following firms is making above normal profit in the short run?A. Image with alt text: A line graph titled "FIRM A" with the y axis labeled "Price or Cost" and the x axis labeled "Quantity". A horizontal line, labeled "Market Price", extends from the center of the y axis. A curved line, labeled "MC" starts at the lower left and goes sharply upward, crossing the center of the Market Price line, ending in the center of the graph. A curved line, labeled "AVC", starts above it near the y axis and extends out, crossing the Market Price line at the far left and the far right of the graph. A third curved line, labeled "ATC" starts high on the left and extends out and to the right, curving just above the Market Price line, and ending between the other two lines.B. Image with alt text: A line graph titled "FIRM B" with the y axis labeled "Price or Cost" and the x axis labeled "Quantity". A horizontal line, labeled "Market Price", extends from the center of the y axis. A curved line, labeled "MC" starts at the lower left and goes sharply upward, crossing the center of the Market Price line, ending in the center of the graph. A curved line, labeled "AVC", starts above it and to the left, near the y axis and extends out, crossing the Market Price line at the far right of the graph. A third curved line, labeled "ATC" starts high on the left and extends out and to the right, crossing the Market Price line at the left and again on the right, and ending between the other two lines.C. Image with alt text: A line graph titled "FIRM C" with the y axis labeled "Price or Cost" and the x axis labeled "Quantity". A horizontal line, labeled "Market Price", extends from the center of the y axis. A curved line, labeled "MC" starts at the lower left and goes sharply upward, crossing the center of the Market Price line, ending in the center of the graph. A curved line, labeled "AVC", starts above it near the y axis and extends out, crossing the Market Price line at the far right of the graph. A third curved line, labeled "ATC" starts high on the left and extends out and to the right, curving just above the Market Price line, and ending between the other two lines.D. Image with alt text: A line graph titled "FIRM D" with the y axis labeled "Price or Cost" and the x axis labeled "Quantity". A horizontal line, labeled "Market Price", extends from the center of the y axis. A curved line, labeled "MC" starts at the lower left and goes sharply upward, crossing the center of the Market Price line, ending in the center of the graph. A curved line, labeled "AVC", starts above it and extends out, curving just above the Market Price line and ending at the far right of the graph. A third curved line, labeled "ATC" starts on the left above the AVC line and extends out and to the right, curving above the Market Price line, and ending between the other two lines. the domestic production of soccer balls willfall to 2 soccer balls, and domestic consumption willrise to 4 soccer balls. therefore, wiknam willimport soccer balls in this case. 6. Armenia had a favorable balance of trade in 2018 when it exported $800 million in goods and services and imported $1.5 billion. What is the generic name of Aloxi? Dolasetron Granisetron Ondanestron Palonosetron Question 7 of 10Which of the following is a way group behavior can help a human communitysurvive?O A. Constructing housingOB. Consuming more productsOC. Growing individual gardensOD. Polluting the water supplySUBMIT Curriculum Compacting is a strategy that works best forA) English Language Learners.B) gifted students.C) Cognitively challenged students Use the graph of the rational function to complete the following statement.As , .Question content area bottom leftPart 1As , enter your response here....Question content area rightPart 1-10-8-6-4-2246810-10-8-6-4-2246810xyA coordinate system has a horizontal x-axis labeled from negative 10 to 10 in increments of 1 and a vertical y-axis labeled from negative 10 to 10 in increments of 1. A graph has three branches and asymptotes y= 1, x = negative 3 and x =3. The first branch is above y equals 1 and to the left of x equals negative 3 comma approaching both. The second branch opens downward between the vertical asymptotes comma reaching a maximum at left parenthesis 0 comma 0 right parenthesis . The third branch is above y equals 1 and to the right of x equals 3 comma approaching both.Asymptotes are shown as dashed lines. The horizontal asymptote is y = 1 The vertical asymptotes are x = -3 and x=3 2. 08 Presentaciones- Asignacin Answer the following in complete Spanish sentences. 1. Quines estn en el muelle (deck)? 2. En qu parte del mundo (world) estn los amigos? 3. Quin va a nadar en la piscina? 4. Qu tiempo hace hoy? 5. Por qu no quiere nadar Susana? 6. Qu van a visitar en Mxico? 7. A qu hora se levanta Pablo maana? 8. Qu hace Pablo por la maana? 9. A qu hora van a desayunar los amigos? 10. Qu se pone Pablo por la maana? The drug used in the management of a patient with acute pulmonary edema that will decrease both preload and afterload and provide relief of anxiety isA. morphine.B. amrinone.C. dobutamine.D. aminophylline. a macromolecule that can exist as linear or cyclized, where it has a 1:2:1 ratio of carbon, hydrogen and oxygen